LSAT and Law School Admissions Forum

Get expert LSAT preparation and law school admissions advice from PowerScore Test Preparation.

 tanushreebansal
  • Posts: 21
  • Joined: Jun 26, 2017
|
#38962
Hi! Can someone explain why answers B and E are incorrect and answer A is correct? I didn't even consider answer A because even if customer don't pay attention to merchandise immediately when they enter the store, that's not to say that they can't pay attention to displays at the front of the store as they walk around. I didn't really think this explained the little difference in sales. Can you help me see why I'm wrong?
 Adam Tyson
PowerScore Staff
  • PowerScore Staff
  • Posts: 5153
  • Joined: Apr 14, 2011
|
#39030
In a Resolve the Paradox question, we aren't necessarily looking for an answer that makes the stimulus perfect, tanushreebansal. Rather, we are looking for an answer which, if true, helps to explain how the supposed paradox might have come to exist, how the two things that don't appear to make much sense can, in fact, make sense.

If it's true that customers don't pay attention right when they walk in, that could explain why they overlooked the hammers on sale, and if they were there to buy a hammer they would have found only the normal-priced hammers. That isn't perfect, as you pointed out - who's to say they don't wander back to the front at some point and see the display? - but perfect isn't needed; help is what's needed, and this answer does help.

Answer B, however, does not help, because up until now customers bought both brands equally. That suggests that customers find the two to be of roughly equal quality and service, doesn't it? If that's so, then why did they suddenly buy fewer of the ones on sale? It would require too much outside help and imagination to say "the low price of the sale hammers made some customers believe that they were of lower quality than the higher priced ones". It's too much for us to read that into the answer - don't help the answer by adding all that back story to it! How else does B explain the non-sale hammers outselling the sale hammers? It doesn't.

Answer E is also no help, because this stimulus has nothing to do with motivating anyone to go to the store. It doesn't matter that the sale didn't bring them in the door, because up until the hammers went on sale, both brands sold about equally. Someone is coming to buy hammers, whether there is a sale or not, so why aren't they buying as many of the sale hammers as they are of the non-sale hammers? How does "I didn't come here just because that hammer is on sale" help explain this odd result? It doesn't.

Don't look for perfect when the stem asks only which one "does most to explain the surprising result". Instead, pick the one that does just that - helps more than the other four answers do. If you read answer A and then you go "oh, yeah, I see how that might help", then you know you have a contender.

Keep pounding, tanushreebansal!
 LSAT1416
  • Posts: 6
  • Joined: May 15, 2018
|
#46037
Hi Adam,

I would like a little bit of further clarification for A and B.

For B, I assumed that roughly equal numbers could make it completely reasonable that Styron slightly outsells Maxlast in any given week. This looked to me like it demonstrated no change in the buying pattern of the public, meaning that the price did not have any impact on their considerations. Am I misinterpreting the meanings of roughly equal or slightly outsells, or is this just a case of not being a strong enough evaluation compared to A?

For A, my issue comes from the word detailed. Just because someone does not take a detailed look wouldn't seem to mean that they don't see anything at all when entering a store. To me, undetailed looks seem likely to have some effect (otherwise why use a word like detailed looks rather than make decisions), and doesn't seem to answer the question.

Am I wrong in interpretation of one or both of these, or am I just not accurately deciding which one is more right?

Thank you
 James Finch
PowerScore Staff
  • PowerScore Staff
  • Posts: 943
  • Joined: Sep 06, 2017
|
#46087
Hi LSAT1416,

The issue with answer choice (B) is that it requires an assumption that Styron's "quality and service" equals or exceeds that of Maxlast. However, because the stimulus gives us no indication of the relative levels of quality and service between the two companies, we can't logically justify that assumption: Maxlast could provide much better quality and service than Styron, and the paradox would remain. When testing an answer choice like this, be sure to always see what it does at both of the logical extremes contained within it (ie Maxlast having much higher quality and service than Styron, or vice versa).

Answer choice (A) works because it deals with one of the two distinctions that the stimulus does give us: product location (the other being price). What (A) is getting at is that, counter-intuitively, sticking the Maxlast hammers close to the store entrance lead shoppers to ignore them, instead going to the usual spot where the store's hammers are located. So (A) can only help explain the paradox, which leads to it being the correct answer choice.

Hope this helps!
 blade21cn
  • Posts: 100
  • Joined: May 21, 2019
|
#65797
Not sure if I can squeeze anything more out of (A) to make a strained explanation.

(A) is qualified by "for the first several seconds after shoppers enter a store." What does that piece of information imply? It's unthinkable for any shopper to complete a transaction or make a purchase decision within that amount of time. Consequently, (A) doesn't tell us what the shoppers do after those initial seconds.

I can comprehend James's suggestion that Maxlast's change of location (though in some people's mind a more favorable location) might potentially hurt its sales, because they are no longer in the usual hammer section at the store, but (A) says "not take detailed notice of the store's merchandise." How is that general language linked to the display case inside the store entrance, where Maxlast is placed? Wouldn't that be "too much outside help and imagination" to make an answer choice work?
 George George
PowerScore Staff
  • PowerScore Staff
  • Posts: 48
  • Joined: Jun 07, 2019
|
#65874
@blade21cn

Great Q! Well, well, well... The LSAT does require some assumptions, now, doesn't it? The LSAT is explicit about this in the Directions, but not explicit enough, perhaps. It says "commonsense" assumptions are fine, but not "superfluous" ones - and the age-old debate begins, particularly in Qs like Resolve the Paradox and Strengthen/Weaken, where the right answer should allude to the connection it has with the stimulus but not always so explicitly. This can be maddening for an LSAT student. After years of pondering this, and doing many LSAT Qs, here's one insight I have: The LSAT assumes capitalism in North America. It assumes you have some experience with capitalism, including profits, investments, commercial transactions, merchandising, etc., and here, e.g. with retail shopping. (This is not unreasonable, except for the fact that the LSAT isn't always explicit about its moves. Don't get "philosophical" on this test too much - pretend you're a cynical worldly lawyer!)

Now, with that assumption in mind, (A) is not so "strained," as you put it, @blade21cn. Obviously, if people don't "take detailed notice" of merchandise, then they wouldn't see that Maxlast hammers are on sale, and then they'd be no more likely to buy them, and that could (though obviously doesn't conclusively) explain why they're not buying Maxlast hammers. The takeaway here is really to look for "Easter eggs" - the LSAT likes to "set up" its own problems in the stimulus as much as possible. The stimulus gave you a clue that the placement of Maxlast hammers might be a material fact when it said "all of the Maxlast hammers were put on sale and placed in a display case just inside the store entrance" and then answer (A) makes use of this fact. When you see answers "dovetail" with stimulus facts, you should look closely because generally the LSAT likes to you "follow its trail." Again, that would be the takeaway for this Q.
 blade21cn
  • Posts: 100
  • Joined: May 21, 2019
|
#65945
Thanks, George. I think I got it now. (A) is linked to "a display case just inside the store entrance" because of "[f]or the first several seconds after shoppers enter a store." I think it's a reasonable assumption that if shoppers ever notice the discounted Maxlast hammers, which were placed near the entrance, they would've noticed them within the first few seconds after they enter the store. Once they walk through that section without paying much attention, they wouldn't be able to see them again elsewhere at the store.

Get the most out of your LSAT Prep Plus subscription.

Analyze and track your performance with our Testing and Analytics Package.